Saltar al contenido

Derivación de la densidad lagrangiana para el campo electromagnético

Bienvenido a nuestro sitio web, en este sitio encontrarás la respuesta de lo que andabas buscando.

Solución:

Abstracto

A continuación, demostraremos que una densidad lagrangiana compatible para el campo electromagnético en el espacio vacío es begin ecuación mathcal L _ em : = : epsilon_ 0 cdot dfrac ! derecha 2 – rho phi + mathbf j boldsymbol cdot mathbf A tag 045 end ecuación que son las ecuaciones de Euler-Langrange producidas a partir de este lagrangiano son las ecuaciones de Maxwell para el campo electromagnético.

Esta densidad lagrangiana se obtiene mediante ensayo y error. (1) procedimiento, no adivinando.

1. Introducción

Las ecuaciones diferenciales de Maxwell del campo electromagnético en el espacio vacío son begin align boldsymbol nabla boldsymbol times mathbf E & = – frac partial mathbf B partial t etiqueta 001a \ boldsymbol nabla boldsymbol times mathbf B & = mu_ 0 mathbf j + frac 1 c ^ 2 frac parcial mathbf E parcial t etiqueta 001b \ nabla boldsymbol cdot mathbf E & = frac rho epsilon_ 0 tag 001c \ nabla boldsymbol cdot mathbf B & = 0 tag 001d end align donde $ : mathbf E = $ vector de intensidad de campo eléctrico , $ : mathbf B = $ vector de densidad de flujo magnético, $ : rho = $ densidad de carga eléctrica, $ : mathbf j = $ vector de densidad de corriente eléctrica. Todas las cantidades son funciones de las tres coordenadas espaciales $ : left (x_ 1, x_ 2, x_ 3 right) equiv left (x, y, z right) : $ y tiempo $ : t equiv x_ 4 : $.

De la ecuación (001d) el vector de flujo magnético $ : mathbf B : $ puede expresarse como la curva de un potencial vectorial $ : mathbf A : $ begin ecuación mathbf B = boldsymbol nabla boldsymbol times mathbf A tag 002 end ecuación y de (002) la ecuación (001a) produce begin ecuación boldsymbol nabla boldsymbol times left ( mathbf E + frac partial mathbf A partial t right) = boldsymbol 0 tag 003 end ecuación por lo que el término entre paréntesis se puede expresar como el gradiente de una función escalar begin ecuación * mathbf E + frac partial mathbf A partid t = – boldsymbol nabla phi end ecuación * que es begin ecuación mathbf E = – boldsymbol nabla phi – frac partial mathbf A partial t tag 004 end ecuación Entonces, las seis variables escalares, los componentes de los vectores $ : mathbf E : $ y $ : mathbf B : $, se pueden expresar como funciones de 4 escalares variables, el potencial escalar $ : phi : $ y tres componentes del potencial vectorial $ : mathbf A $.

Insertando las expresiones de $ : mathbf E : $ y $ : mathbf B : $, ecuaciones (002) y (004) respectivamente, en las ecuaciones (001b) y (001c) tenemos
begin ecuación boldsymbol nabla boldsymbol times left ( boldsymbol nabla boldsymbol times mathbf A right) = mu_ 0 mathbf j + frac 1 c ^ 2 frac parcial parcial t izquierda (- boldsymbol nabla phi – frac parcial mathbf A parcial t derecha) etiqueta 005 end ecuación
y begin ecuación boxed : – nabla ^ 2 phi- frac partial partial t left ( nabla boldsymbol cdot mathbf A right ) = frac rho epsilon_ 0 : tag 006 end ecuación Dado que begin ecuación boldsymbol nabla boldsymbol times left ( boldsymbol nabla boldsymbol times mathbf A right) = boldsymbol nabla left ( nabla boldsymbol cdot mathbf A right) – nabla ^ 2 mathbf A tag 007 end ecuación ecuación (005) produce begin ecuación boxed : frac 1 c ^ 2 frac parcial ^ 2 mathbf A t parcial ^ 2 – nabla ^ 2 mathbf A + boldsymbol nabla left ( nabla boldsymbol cdot mathbf A + frac 1 c ^ 2 frac parcial phi parcial t right) = mu_ 0 mathbf j : tag 008 end ecuación

2. Las ecuaciones de Euler-Lagrange de EM Field

Ahora, nuestra tarea principal es encontrar una densidad lagrangiana $ : mathcal L : $, función de las cuatro ” coordenadas de campo ” y sus derivadas de primer orden
begin ecuación mathcal L = mathcal L left ( eta _ jmath, overset centerdot eta _ jmath, boldsymbol nabla eta_ jmath right) qquad left ( jmath = 1,2,3,4 right) tag 009 end ecuación tal que las cuatro ecuaciones escalares del campo electromagnético (006) y (008) se derivan de las ecuaciones de Lagrange begin ecuación frac parcial parcial t left[fracpartial mathcalLpartial left(dfracpartial eta_jmathpartial tright)right]+ suma_ k = 1 ^ k = 3 frac parcial parcial x_ k izquierda[fracpartial mathcalLpartial left(dfracpartial eta_jmathpartial x_kright)right]- frac parcial mathcal L parcial eta _ jmath = 0 :, quad left ( jmath = 1,2,3,4 right) tag 010 end ecuación simplificado en notación a begin ecuación boxed : dfrac parcial parcial t left ( dfrac parcial mathcal L parcial overset centerdot eta _ jmath right) + nabla boldsymbol cdot left[dfracpartial mathcalLpartial left(boldsymbolnablaeta_jmathright)right]- frac parcial mathcal L parcial eta _ jmath = 0, quad left ( jmath = 1,2,3,4 right) : tag 011 end ecuación

Aquí la densidad lagrangiana $ : mathcal L : $ es una función de

  1. las cuatro ” coordenadas de campo ”

begin align eta_ 1 & = mathrm A _1 left (x_ 1, x_ 2, x_ 3, t right) tag 012.1 \ eta_ 2 & = mathrm A _2 left (x_ 1, x_ 2, x_ 3, t right) tag 012.2 \ eta_ 3 & = mathrm A _3 left (x_ 1, x_ 2, x_ 3, t right) tag 012.3 \ eta_ 4 & = : ; phi left (x_ 1 , x_ 2, x_ 3, t right) tag 012.4 end align

  1. sus derivados de tiempo

begin align overset centerdot eta _ 1 & equiv dfrac partial eta_ 1 partial t = dfrac partial mathrm A _ 1 parcial t equiv overset centerdot mathrm A _ 1 tag 013.1 \ overset centerdot eta _ 2 & equiv dfrac parcial eta_ 2 parcial t = dfrac parcial mathrm A _ 2 parcial t equiv overset centerdot mathrm A _ 2 etiqueta 013.2 \ overset centerdot eta _ 3 & equiv dfrac partial eta_ 3 partial t = dfrac parcial mathrm A _ 3 parcial t equiv overset centerdot mathrm A _ 3 tag 013.3 \ overset centerdot eta _ 4 & equiv dfrac parcial eta_ 4 parcial t = dfrac parcial phi parcial t equiv overset centerdot phi etiqueta 013.4 end align

y

  1. sus gradientes

begin ecuación begin array cccc boldsymbol nabla eta_ 1 = boldsymbol nabla mathrm A _1 :, : & boldsymbol nabla eta_ 2 = boldsymbol nabla mathrm A _ 2 :, : boldsymbol nabla eta_ 3 = boldsymbol nabla mathrm A _3 :, : & boldsymbol nabla eta_ 4 = boldsymbol nabla phi end matriz etiqueta 014 end ecuación

Expresamos las ecuaciones (006) y (008) en formas que son similares a las ecuaciones de Lagrange (011). cdot mathbf A right) + nabla boldsymbol cdot left ( boldsymbol nabla phi right) – left (- frac rho epsilon_ 0 derecha) = 0 :} etiqueta 015 end ecuación y begin ecuación recuadro : dfrac parcial parcial t izquierda ( frac parcial mathrm A _ k parcial t + frac parcial phi parcial x_ k derecha) + nabla boldsymbol cdot izquierda[c^2left(fracpartial mathbfApartial x_k- boldsymbolnablamathrmA_kright)right] – frac mathrm j _ k epsilon_ 0 = 0 : tag 016 end ecuación La ecuación de Lagrange (011) para $ : jmath = 4 : $, eso es para $ : eta_ 4 = phi : $, es begin ecuación frac parcial parcial t left ( frac parcial mathcal L parcial overset centerdot phi right) + nabla boldsymbol cdot left[fracpartial mathcalLpartial left(boldsymbolnablaphiright)right]- frac parcial mathcal L parcial phi = 0 etiqueta 017 end ecuación

Comparando las ecuaciones (015) y (017), observamos que la primera podría derivarse de la segunda si begin equal dfrac partial mathcal L partial overset centerdot phi = nabla boldsymbol cdot mathbf A :, qquad dfrac partial mathcal L partial left ( boldsymbol nabla phi right) = boldsymbol nabla phi :, qquad frac partial mathcal L partial phi = – frac rho epsilon_ 0 tag 018 end ecuación
de modo que la densidad lagrangiana $ : mathcal L : $ debe contener respectivamente los términos begin ecuación mathcal L _ boldsymbol alpha_ 1 equiv left ( nabla boldsymbol cdot mathbf A right) overset centerdot phi :, qquad mathcal L _ boldsymbol alpha_ 2 equiv tfrac 1 2 Vert boldsymbol nabla phi Vert ^ 2 :, qquad mathcal L _ boldsymbol alpha_ 3 equiv- frac rho phi epsilon_ 0 etiqueta 019 end ecuación y, en consecuencia, su suma begin ecuación mathcal L _ boldsymbol alpha = mathcal L _ boldsymbol alpha_ 1 + mathcal L _ boldsymbol alpha_ 2 + mathcal L _ boldsymbol alpha_ 3 = left ( nabla boldsymbol cdot mathbf A right) overset centerdot phi + tfrac 1 2 Vert boldsymbol nabla phi Vert ^ 2 – frac rho phi epsilon_ 0 etiqueta 020 end ecuación

Suponemos que una densidad lagrangiana apropiada $ : mathcal L : $ sería de la forma begin ecuación mathcal L = mathcal L _ boldsymbol alpha + mathcal L _ boldsymbol beta tag 021 end ecuación y desde $ : mathcal L _ boldsymbol alpha : $ produce la ecuación (015) , esperamos que $ : mathcal L _ boldsymbol beta : $, por determinar, producirá las ecuaciones (016). Esta expectativa sería correcta si las ecuaciones (015) y (016) estuvieran desacopladas, por ejemplo, si la primera contiene solo términos $ : phi $ y la segunda solo términos $ : mathbf A $. Pero aquí este no es el caso: $ : mathcal L _ boldsymbol alpha : $ como conteniendo $ : mathbf A $ -terms participarían en la producción de ecuaciones (016 ) y además $ : mathcal L _ boldsymbol beta : $ participaría en la producción de la ecuación (015), posiblemente destruyendo mutuamente la producción de las ecuaciones como esperábamos. Pero aquí seguimos un procedimiento de prueba y error, que nos dirigirá a la respuesta correcta como veremos a continuación.

Ahora, las ecuaciones de Lagrange (011) para $ : jmath = k = 1,2,3 : $, eso es para $ : eta_ k = mathrm A _ k : $ , son begin ecuación frac parcial parcial t izquierda ( dfrac parcial mathcal L parcial desbordamiento centerdot mathrm A _ k derecha) + nabla boldsymbol cdot izquierda[dfracpartial mathcalLpartial left(boldsymbolnablamathrmA_kright)right]- frac parcial mathcal L parcial mathrm A _ k = 0 etiqueta 022 end ecuación

Al comparar las ecuaciones (016) y (022), notamos que la primera podría derivarse de la segunda si begin equal dfrac partial mathcal L partial overset centerdot mathrm A _ k = overset centerdot mathrm A _ k + frac partial phi partial x_ k :, quad dfrac parcial mathcal L parcial izquierda ( boldsymbol nabla mathrm A _ k derecha) = c ^ 2 izquierda ( frac parcial mathbf A parcial x_ k – boldsymbol nabla mathrm A _ k derecha) :, quad frac parcial mathcal L parcial mathrm A _ k = frac mathrm j _ k epsilon_ 0 etiqueta 023 end ecuación

De la primera de las ecuaciones (023) la $ : mathcal L _ boldsymbol beta : $ parte del La densidad de Lagrange $ : mathcal L : $ debe contener los términos begin ecuación frac 1 2 left Vert overset centerdot mathrm A _ k right Vert ^ 2 + frac partial phi partial x_ k overset centerdot mathrm A _ k :, quad k = 1, 2, 3 etiqueta 024 end ecuación y, por lo tanto, su suma con respecto a $ : k : $ begin ecuación mathcal L _ boldsymbol beta_ 1 equiv tfrac 1 2 left Vert mathbf dot A right Vert ^ 2 + boldsymbol nabla phi boldsymbol cdot mathbf dot A etiqueta 025 end ecuación

A partir de la segunda de las ecuaciones (023), la $ : mathcal L _ boldsymbol beta : $ parte de la densidad de Lagrange $ : mathcal L : $ debe contener los términos comenzar ecuación tfrac 1 2 c ^ 2 left[fracpartial mathbfApartial x_k boldsymbolcdot boldsymbolnablamathrmA_k -Vert boldsymbolnablamathrmA_kVert^2right] :, quad k = 1,2,3 tag 026 end ecuación y, por tanto, su suma con respecto a $ : k : $ begin ecuación mathcal L _ boldsymbol beta_ 2 equiv tfrac 1 2 c ^ 2 sum ^ k = 3 _ k = 1 left[ fracpartial mathbfApartial x_kboldsymbolcdot boldsymbolnablamathrmA_k-VertboldsymbolnablamathrmA_kVert^2right]
tag 027 end ecuación De la tercera de las ecuaciones (023) la $ : mathcal L _ boldsymbol beta : $ parte de la densidad de Lagrange $ : mathcal L : $ debe contener los términos begin ecuación frac mathrm j _ k mathrm A _ k epsilon_ 0 :, quad k = 1,2,3 etiqueta 028 end ecuación y, por tanto, su suma con respecto a $ : k : $ begin ecuación mathcal L _ boldsymbol beta_ 3 equiv frac mathbf j boldsymbol cdot mathbf A epsilon_ 0 tag 029 end ecuación

De las ecuaciones (025), (027) y (029) la $ : mathcal L _ boldsymbol beta : $ parte de la densidad de Lagrange $ : mathcal L : $ es begin align mathcal L _ boldsymbol beta & = mathcal L _ boldsymbol beta_ 1 + mathcal L _ boldsymbol beta_ 2 + mathcal L _ boldsymbol beta_ 3 tag 030 \ & = tfrac 1 2 left Vert mathbf punto A right Vert ^ 2 + boldsymbol nabla phi boldsymbol cdot mathbf dot A + tfrac 1 2 c ^ 2 sum ^ k = 3 _ k = 1 left[ fracpartial mathbfApartial x_k boldsymbolcdot boldsymbolnablamathrmA_k-VertboldsymbolnablamathrmA_kVert^2right]+ frac mathbf j boldsymbol cdot mathbf A epsilon_ 0 nonumber end align

Finalmente, de las expresiones (020) y (030) para las densidades $ : mathcal L _ boldsymbol alpha, mathcal L _ boldsymbol beta : $ la densidad de Lagrange $ : mathcal L = mathcal L _ boldsymbol alpha + mathcal L _ boldsymbol beta : $ es begin align mathcal L & = mathcal L _ boldsymbol alpha + mathcal L _ boldsymbol beta tag 031 \ & = left ( nabla boldsymbol cdot mathbf A right) overset centerdot phi + tfrac 1 2 Vert boldsymbol nabla phi Vert ^ 2 – frac rho phi epsilon_ 0 nonumber \ & + tfrac 1 2 left Vert mathbf dot A right Vert ^ 2 + boldsymbol nabla phi boldsymbol cdot mathbf dot A + tfrac 1 2 c ^ 2 sum ^ k = 3 _ k = 1 izquierda[ fracpartial mathbfApartial x_k boldsymbolcdot boldsymbolnablamathrmA_k-VertboldsymbolnablamathrmA_kVert^2right]+ frac mathbf j boldsymbol cdot mathbf A epsilon_ 0 nonumber \ & text (esta es una densidad de Lagrange incorrecta) nonumber end alinear

3. Error-Prueba-Éxito final

La inserción de esta expresión de densidad de Lagrange en la ecuación de Lagrange con respecto a $ : phi : $, que es la ecuación (017), no produce la ecuación (006) pero
begin ecuación – nabla ^ 2 phi- frac parcial parcial t left (2 nabla boldsymbol cdot mathbf A right) = frac rho epsilon_ 0 :, quad ( textbf incorrecto) etiqueta 032 end ecuación La aparición de un $ : left ( nabla boldsymbol cdot mathbf A right) : $ se debe al término $ : left ( boldsymbol nabla phi boldsymbol cdot mathbf dot A right) : $ de $ : mathcal L _ boldsymbol beta : $ y es por eso que la densidad de Lagrange dada por la ecuación (031) no es apropiada.

Para resolver este problema debemos mirar (015), es decir (006), desde un punto de vista diferente de la siguiente manera begin ecuación nabla boldsymbol cdot left ( boldsymbol nabla phi + mathbf dot A right) – left (- frac rho epsilon_ 0 right) = 0 tag 033 end ecuación

Comparando las ecuaciones (033) y (017), observamos que la primera podría derivarse de la segunda si en lugar de (018) tenemos
begin ecuación dfrac parcial mathcal L parcial desbordamiento centerdot phi = 0 :, qquad dfrac parcial mathcal L parcial izquierda ( boldsymbol nabla phi right) = boldsymbol nabla phi + mathbf dot A :, qquad frac parcial mathcal L parcial phi = – frac rho epsilon_ 0 etiqueta 034 end ecuación
así que en lugar de (019) y (020) respectivamente las ecuaciones begin ecuación mathcal L ^ prime _ boldsymbol alpha_ 1 equiv 0 :, quad mathcal L ^ prime _ boldsymbol alpha_ 2 equiv tfrac 1 2 Vert boldsymbol nabla phi Vert ^ 2 + boldsymbol nabla phi boldsymbol cdot mathbf dot A :, quad mathcal L ^ prime _ boldsymbol alpha_ 3 = mathcal L _ boldsymbol alpha_ 3 equiv- frac rho phi epsilon_ 0 tag 035 end ecuación comenzar ecuación mathcal L ^ prime _ boldsymbol alpha = mathcal L ^ prime _ boldsymbol alpha_ 1 + mathcal L ^ prime _ boldsymbol alpha_ 2 + mathcal L ^ prime _ boldsymbol alpha_ 3 = tfrac 1 2 Vert boldsymbol nabla phi Vert ^ 2 + boldsymbol nabla phi boldsymbol cdot mathbf dot A – frac rho phi epsilon_ 0 etiqueta 036 end ecuación Ahora, es necesario omitir de $ : mathcal L _ boldsymbol beta_ 1 : $, ecuación (025), el segundo término $ : left ( boldsymbol nabla phi boldsymbol cdot mathbf dot A right) : $ ya que aparece en $ : mathcal L ^ prime _ boldsymbol alpha_ 2 : $, mira el segundo de arriba ecuaciones (035).

Así que tenemos en lugar de (025) begin ecuación mathcal L ^ prime _ boldsymbol beta_ 1 equiv tfrac 1 2 left Vert mathbf dot A right Vert ^ 2 tag 037 end ecuación mientras que $ : mathcal L _ boldsymbol beta_ 2, mathcal L _ boldsymbol beta_ 3 : $ permanecen sin cambios como en las ecuaciones (027) y (029) begin align mathcal L ^ prime _ boldsymbol beta_ 2 & = mathcal L _ boldsymbol beta_ 2 equiv tfrac 1 2 c ^ 2 sum ^ k = 3 _ k = 1 left[ fracpartial mathbfApartial x_kboldsymbolcdotboldsymbolnablamathrmA_k-VertboldsymbolnablamathrmA_kVert^2right]
tag 038 \ mathcal L ^ prime _ boldsymbol beta_ 3 & = mathcal L _ boldsymbol beta_ 3 equiv frac mathbf j boldsymbol cdot mathbf A epsilon_ 0 tag 039 end align

En lugar de (030) begin align mathcal L ^ prime _ boldsymbol beta & = mathcal L ^ prime _ boldsymbol beta_ 1 + mathcal L ^ prime _ boldsymbol beta_ 2 + mathcal L ^ prime _ boldsymbol beta_ 3 tag 040 \ & = tfrac 1 2 left Vert mathbf dot A right Vert ^ 2 + tfrac 1 2 c ^ 2 sum ^ k = 3 _ k = 1 left[ fracpartial mathbfApartial x_k boldsymbolcdot boldsymbolnablamathrmA_k-VertboldsymbolnablamathrmA_kVert^2right]+ frac mathbf j boldsymbol cdot mathbf A epsilon_ 0 nonumber end align y finalmente para la nueva densidad lagrangiana que tenemos en lugar de (031)

begin align mathcal L ^ prime & = mathcal L ^ prime _ boldsymbol alpha + mathcal L ^ prime _ boldsymbol beta tag 041 \ & = tfrac 1 2 Vert boldsymbol nabla phi Vert ^ 2 + boldsymbol nabla phi boldsymbol cdot mathbf dot A – frac rho phi epsilon_ 0 nonumber \ & + tfrac 1 2 left Vert mathbf dot A right Vert ^ 2 + tfrac 1 2 c ^ 2 sum ^ k = 3 _ k = 1 left[ fracpartial mathbfApartial x_k boldsymbolcdot boldsymbolnablamathrmA_k -VertboldsymbolnablamathrmA_kVert^2right]+ frac mathbf j boldsymbol cdot mathbf A epsilon_ 0 nonumber end align

La densidad $ : mathcal L ^ prime : $ de (041) se obtiene de la densidad $ : mathcal L : $ de (031) si omitimos el término $ : left ( nabla boldsymbol cdot mathbf A right) overset centerdot phi : $. Entonces $ : mathcal L ^ prime : $ es independiente de $ : overset centerdot phi $.

En las siguientes ecuaciones, la llave sobre los 3 términos de la izquierda agrupa la parte de la densidad $ : mathcal L ^ prime : $ que esencialmente participa en la producción de la ecuación electromagnética (006) a partir de la ecuación de Lagrange. con respecto a $ : phi : $, ecuación (017), mientras que la llave debajo de los 4 términos correctos agrupa esa parte de la densidad $ : mathcal L ^ prime : $ que esencialmente participa a la producción de las ecuaciones electromagnéticas (008) a partir de las ecuaciones de Lagrange con respecto a $ : mathrm A _ 1, mathrm A _ 2, mathrm A _ 3 : $, ecuación (022).

begin ecuación * mathcal L ^ prime = overbrace tfrac 1 2 Vert boldsymbol nabla phi Vert ^ 2 – frac rho phi epsilon_ 0 + boldsymbol nabla phi boldsymbol cdot mathbf dot A ^ text con respecto a phi + tfrac 1 2 left Vert mathbf dot A right Vert ^ 2 + tfrac 1 2 c ^ 2 sum ^ k = 3 _ k = 1 left[fracpartial mathbfApartial x_k boldsymbolcdot boldsymbolnablamathrmA_k-Vert boldsymbolnablamathrmA_kVert^2right]+ frac mathbf j boldsymbol cdot mathbf A epsilon_ 0 end ecuación *

begin ecuación * mathcal L ^ prime = tfrac 1 2 Vert boldsymbol nabla phi Vert ^ 2 – frac rho phi epsilon_ 0 + underbrace boldsymbol nabla phi boldsymbol cdot mathbf dot A + tfrac 1 2 left Vert mathbf dot A right Vert ^ 2 + tfrac 1 2 c ^ 2 sum ^ k = 3 _ k = 1 left[fracpartial mathbfApartial x_k boldsymbolcdot boldsymbolnablamathrmA_k-VertboldsymbolnablamathrmA_kVert^2right]+ frac mathbf j boldsymbol cdot mathbf A epsilon_ 0 _ text con respecto a mathbf A end ecuación *

Tenga en cuenta el término común $ : left ( boldsymbol nabla phi boldsymbol cdot mathbf dot A right) $.

Al reordenar los términos en la expresión (041) de la densidad $ : mathcal L ^ prime : $ tenemos begin ecuación mathcal L ^ prime = underbrace tfrac 1 2 left Vert mathbf dot A right Vert ^ 2 + tfrac 1 2 Vert boldsymbol nabla phi Vert ^ 2 + boldsymbol nabla phi boldsymbol cdot mathbf dot A _ tfrac 1 2 left Vert – boldsymbol nabla phi – frac parcial mathbf A parcial t right Vert ^ 2 – tfrac 1 2 c ^ 2 underbrace sum ^ k = 3 _ k = 1 left[Vert boldsymbolnablamathrmA_kVert^2-fracpartial mathbfApartial x_kboldsymbolcdot boldsymbolnablamathrmA_kright] _ left Vert boldsymbol nabla boldsymbol times mathbf A right Vert ^ 2 + frac 1 epsilon_ 0 left (- rho phi + mathbf j boldsymbol cdot mathbf A right) end ecuación begin ecuación —————- – etiqueta 042 end ecuación

es decir, begin ecuación mathcal L ^ prime = tfrac 1 2 left | ! left | – boldsymbol nabla phi – frac partial mathbf A t parcial right | ! right | ^ 2 – tfrac 1 2 c ^ 2 left | ! left | boldsymbol nabla boldsymbol times mathbf A right | ! right | ^ 2 + frac 1 epsilon_ 0 left (- rho phi + mathbf j boldsymbol cdot mathbf A derecha) etiqueta 043 end ecuación o begin ecuación mathcal L ^ prime = frac ! left 2 + frac 1 epsilon_ 0 left (- rho phi + mathbf j boldsymbol cdot mathbf A right ) etiqueta 044 end ecuación

Ahora, si la densidad $ : mathcal L ^ prime : $ debe tener dimensiones de energía por unidad de volumen, definimos $ : mathcal L _ em = epsilon_ 0 mathcal L ^ prime : $ entonces begin ecuación boxed : mathcal L _ em : = : epsilon_ 0 cdot dfrac ! right 2 – rho phi + mathbf j boldsymbol cdot mathbf A : tag 045 end ecuación teniendo en cuenta que begin align left Vert mathbf E right Vert ^ 2 & = left Vert – boldsymbol nabla phi – dfrac partial mathbf A t parcial right Vert ^ 2 = left Vert mathbf dot A right Vert ^ 2 + Vert boldsymbol nabla phi Vert ^ 2 +2 left ( boldsymbol nabla phi boldsymbol cdot mathbf dot A right) tag 046a \ & nonumber \ left Vert mathbf B right Vert ^ 2 & = left Vert boldsymbol nabla boldsymbol times mathbf A right Vert ^ 2 = sum ^ k = 3 _ k = 1 left[Vert boldsymbolnablamathrmA_kVert^2-dfracpartial mathbfApartial x_kboldsymbolcdot boldsymbolnablamathrmA_kright]
etiqueta 046b end align

El escalar $ : left ( left | ! Left | mathbf E right | ! Right | ^ 2 -c ^ 2 left | ! Left | mathbf B right | ! Right | ^ 2 right) : $ es uno de los dos invariantes de Lorentz (2) del campo (el otro es $ : mathbf E boldsymbol cdot mathbf B $) esencialmente igual a una constante por $ : mathcal E _ mu nu mathcal E ^ mu nu : $, donde $ : mathcal E ^ mu nu : $ el campo antisimétrico(2) tensor.

Por otro lado, el escalar $ : left (- rho phi + mathbf j boldsymbol cdot mathbf A right) : $ es esencialmente el producto interno $ : J_ mu A ^ mu : $ en el espacio de Minkowski de dos 4 vectores: la densidad de 4 corrientes $ : J ^ mu = left (c rho, mathbf j derecha) : $ y el 4-potencial $ : A ^ mu = left ( phi / c, mathbf A right) : $, un escalar invariante de Lorentz también.

Entonces, la densidad de Lagrange $ : mathcal L _ em : $ en la ecuación (045) es invariante de Lorentz.


(1) Mediante un procedimiento de prueba y error encontré al Lagrangiano en un caso más difícil y complicado: vea mi respuesta como user82794 aquí Obtenga el lagrangiano del sistema de ecuación acoplada

(2) Siguiendo a W.Rindler en “Introducción a la relatividad especial” Ed.1982, este tensor se deriva en la ecuación (38.15) begin ecuación mathcal E _ mu nu = begin bmatrix 0 & E_ 1 & E_ 2 & E_ 3 \ -E_ 1 & 0 & -cB_ 3 & cB_ 2 \ -E_ 2 & cB_ 3 & 0 & -cB_ 1 \ -E_ 3 & -cB_ 2 & cB_ 1 & 0 end bmatrix quad text so quad mathcal E ^ mu nu = begin bmatrix 0 & -E_ 1 & – E_ 2 & -E_ 3 \ E_ 1 & 0 & -cB_ 3 & cB_ 2 \ E_ 2 & cB_ 3 & 0 & -cB_ 1 \ E_ 3 & -cB_ 2 & cB_ 1 & 0 end bmatrix tag 38.15 end ecuación que al hacer los reemplazos (de dualidad) $ : mathbf E to -c mathbf B : $ y $ : c mathbf B to mathbf E : $ produce begin ecuación mathcal B _ mu nu = begin bmatrix 0 & -cB_ 1 & -cB_ 2 & – cB_ 3 \ cB_ 1 & 0 & -E_ 3 & E_ 2 \ cB_ 2 & cE_ 3 & 0 & -E_ 1 \ cB_ 3 & -E_ 2 & E_ 1 & 0 end bmatrix quad text so quad mathcal B ^ mu nu = begin bmatrix 0 & cB_ 1 & cB_ 2 & cB_ 3 \ -cB_ 1 & 0 & -E_ 3 & E_ 2 \ -cB_ 2 & cE_ 3 & 0 & -E_ 1 \ -cB_ 3 & -E_ 2 & E_ 1 & 0 end bmatrix tag 39.05 end ecuación
Las dos invariantes de $ : mathcal E ^ mu nu : $ – inmediatamente reconocibles como tales por su modo de formación – se pueden expresar de la siguiente manera:
begin align X & = dfrac 1 2 mathcal E _ mu nu mathcal E ^ mu nu = – dfrac 1 2 mathcal B _ mu nu mathcal B ^ mu nu = c ^ 2 left | ! left | mathbf B right | ! right | ^ 2 – left | ! Left | mathbf E right | ! Right | ^ 2 tag 39.06 \ Y & = dfrac 1 4 mathcal B _ mu nu mathcal E ^ mu nu = c mathbf B boldsymbol cdot mathbf E tag 39.07 end align

En última instancia, el razonamiento debe ser que (como dijiste) debe construirse de manera que las ecuaciones de Euler-Lagrange sean ecuaciones de Maxwell. Entonces, en cierto sentido, debe adivinar el Lagrangiano que produce esto, como se hace aquí, por ejemplo.

Sin embargo, puede obtener alguna orientación del hecho de que necesitamos construir un Lagrangiano para un campo sin masa sin interacción propia. Entonces necesitamos una combinación invariante de gauge y lorentz del potencial de 4 vectores que solo tiene un término cinético (cuadrático en derivadas de los campos). Entonces no le quedan muchas opciones aparte de $ F ^ mu nu F _ mu nu $. El término fuente es trivial para agregarlo si es necesario.

Estoy casi 100% seguro de que el lagrangiano es una suposición de la teoría. No se puede derivar. No tengo ninguna referencia para esta afirmación. Solo sé que de cada curso que me han enseñado y de cada libro que he leído, el Lagrangiano (asumiendo que se esté utilizando) es donde empiezas. Es el “primer principio” en este caso.

¡Haz clic para puntuar esta entrada!
(Votos: 0 Promedio: 0)



Utiliza Nuestro Buscador

Deja una respuesta

Tu dirección de correo electrónico no será publicada. Los campos obligatorios están marcados con *